Difference between revisions of "2011 AMC 12B Problems/Problem 1"

(Created page with '== Problem == What is <math>(2+4+6)/(1+3+5) - (1+3+5)/(2+4+6)?</math> <math> \textbf{(A)}\ -1 \qquad \textbf{(B)}\ 5/36 \qquad \textbf{(C)}\ 7/12 \qquad \textbf{(D)}\ 147/60 \q…')
 
Line 1: Line 1:
 
== Problem ==
 
== Problem ==
What is <math>(2+4+6)/(1+3+5) - (1+3+5)/(2+4+6)?</math>
+
What is <math>\frac{2+4+6}{1+3+5} - \frac{1+3+5}{2+4+6}?</math>
 
   
 
   
 
<math>
 
<math>
 
\textbf{(A)}\ -1 \qquad
 
\textbf{(A)}\ -1 \qquad
\textbf{(B)}\ 5/36 \qquad
+
\textbf{(B)}\ \frac{5}{36} \qquad
\textbf{(C)}\ 7/12 \qquad
+
\textbf{(C)}\ \frac{7}{12} \qquad
\textbf{(D)}\ 147/60 \qquad
+
\textbf{(D)}\ \frac{147}{60} \qquad
\textbf{(E)}\ 43/3 </math>
+
\textbf{(E)}\ \frac{43}{3} </math>
  
  

Revision as of 15:53, 27 February 2011

Problem

What is $\frac{2+4+6}{1+3+5} - \frac{1+3+5}{2+4+6}?$

$\textbf{(A)}\ -1 \qquad \textbf{(B)}\ \frac{5}{36} \qquad \textbf{(C)}\ \frac{7}{12} \qquad \textbf{(D)}\ \frac{147}{60} \qquad \textbf{(E)}\ \frac{43}{3}$


Solution

See also

2011 AMC 12A (ProblemsAnswer KeyResources)
Preceded by
First Problem
Followed by
Problem 2
1 2 3 4 5 6 7 8 9 10 11 12 13 14 15 16 17 18 19 20 21 22 23 24 25
All AMC 12 Problems and Solutions